0 Daumen
366 Aufrufe

Ich möchte diese Gleichung zeigen:

$$ { (U\cap W) }^{ \bot  }\quad \subset \quad { U }^{ \bot  }\quad +\quad { W }^{ \bot  }\quad $$

Ich schreib mal meine Lösung hier hin

$$ x\quad \in \quad { (U\cap W) }^{ \bot  }\quad =>\quad \exists \quad x\bot u\quad und\quad x\bot w\quad u\in U\quad w\in W $$

$$ =>\quad x\quad \in \quad { U }_{ 1 }^{ \bot  }\quad und\quad x\quad \in \quad { W }_{ 1 }^{ \bot  }\quad für\quad { U }_{ 1 }\subset \quad U\quad { W }_{ 1 }\subset \quad W $$


$$=>\quad x\quad \in \quad { U }_{ 1 }^{ \bot  }\quad +\quad { W }_{ 1 }^{ \bot  }\quad \subset \quad { U }^{ \bot  }+{ W }^{ \bot  } $$


Stimmt das so oder hab ich nen Fehler gemacht?

Avatar von

Ein anderes Problem?

Stell deine Frage

Willkommen bei der Mathelounge! Stell deine Frage einfach und kostenlos

x
Made by a lovely community